Site Loader

Содержание

| Цепь переменного тока с активным и индуктивным сопротивлениями

Из сказанного можно сделать вывод, что емкостное сопротивление обратно пропорционально емкости.

Таким образом. емкостное сопротивление:

где Хс — емкостное сопротивление, ом;

ώ — угловая частота переменного тока, рад/сек;

С —емкость, ф.

Известно, что угловая частота . Поэтому емкостное сопротивление можно определить так:

где f— частота переменного тока, гц.

Если включенная емкость измеряется в микрофарадах, то ем­костное сопротивление

Если емкость измеряется в пикофарадах, то

Следует подчеркнуть, что имеется существенное различие меж­ду емкостным и активным сопротивлениями. Как известно, актив­ная нагрузка безвозвратно потребляет энергию генератора пере­менного тока.

Если же к источнику переменного тока присоединена емкость, то, как было рассмотрено выше, энергия генератора расходуется при заряде конденсатора на создание электрического поля между пластинами и возвращается обратно генератору при разряде кон­денсатора.

Следовательно, емкостная нагрузка не потребляет энергию ге­нератора, а в цепи с емкостью происходит «перекачивание» энергии из генератора в конденсатор и обратно. По этой причине емкостное сопротивление, как и индуктивное, называется реактивным.

Пример. Конденсатор емкостью С=2 мкф включен в цепь переменного тока, частота которого 50 гц. Определить:

1)  его емкостное сопротивление при частоте f=50 гц;

2)  емкостное сопротивление этого конденсатора переменному току, частота которого 500 гц.

Решение. Емкостное сопротивление конденсатора переменному току при частоте f=50 гц

При частоте f=500 гц

 Из приведенного примера видно, что емкостное сопротивление конденсатора уменьшается с повышением частоты, а с уменьшением частоты переменного тока емкостное сопротивление возрастает. Для постоянного тока, когда напряжение на зажимах цепи не изме­няется, конденсатор практически обладает бесконечно большим со­противлением и поэтому он постоянного тока не пропускает.

§ 55. ЦЕПЬ ПЕРЕМЕННОГО ТОКА С АКТИВНЫМ И ИНДУКТИВНЫМ СОПРОТИВЛЕНИЯМИ

Любая проволочная катушка, включенная в цепь переменного тока, обладает активным сопротивлением, зависящим от материала, Длины и сечения проволоки  ,  и индуктивным сопротивлением, которое зависит от  индуктивности катушки и частоты переменного тока, протекающего по ней (ХL=ωL=2

πfL). Такую катушку можно рассматривать как приемник энергии, в котором активное и индуктивное сопротивления соединены последовательно.

Рассмотрим цепь переменного тока, в которую включена катуш­ка индуктивности (рис. 56, а) с активным r и индуктивным ХL со­противлением. Падение напряжения на  активном сопротивлении

Падение напряжения на индуктивном сопротивлении

Построим векторную диаграмму тока и напряжения (рис. 56, б) для рассматриваемой цепи.

Отложим по горизонтали вектор тока  I в выбранном масштабе. Известно, что ток  и напряжение в цепи с активным сопротивлением совпадают по фазе, поэтому  вектор падения напряжения на активном сопротивлении откладываем по вектору  тока.

В цепи с индуктивностью ток отстает от напряжения на угол =90°.  Поэтому вектор падения напряжения на индуктивном сопротивлении откладываем на диаграмме вверх под углом 90° к  вектору тока.

Для определения общего напряжения, приложенного к цепи, сложим векторы  Ua. и UL Суммой этих векторов будет диагональ параллелограмма — вектор  U. Треугольник АОБ, стороны которого  выражают соответственно напряжения Uа, UL и общее напряжение и, называется треугольником напряжений. На основании теоремы Пифагора — в прямоугольном треугольнике квадрат гипотенузы равен сумме квадратов катетов — следует, что общее напряжение на зажимах цепи

Пример. Падение напряжения на активном сопротивлении Ua=15в. Напря­жение  на индуктивном сопротивлении UL=26 в. Вычислить общее напряжение, приложенное к цепи.

Решение. Общее напряжение на зажимах цепи переменного тока с после­довательно соединенными активным и индуктивным сопротивлениями.

Чтобы определить полное сопротивление цепи переменного тока с активным и индуктивным сопротивлениями, следует разделить векторы Ua=Ir и UL =IXL на число I, выражающее силу тока в цепи, и построить треугольник А’О’В’ (рис. 56, в), стороны которого меньше сторон треугольника напряжений в I раз. Образованный треугольник называется треугольником сопротивлений. Его сторо­нами являются сопротивления r и ХL и полное сопротивление цепи Z.

Пользуясь теоремой Пифагора, можно написать что,

отсюда полное сопротивление цепи

Пример. Активное сопротивление катушки r=7 ом, а ее индуктивное сопро­тивление ХL=24 ом. Вычислить полное сопротивление катушки.

Решение. Полное сопротивление катушки переменному току

Сила тока в цепи с  активным и  индуктивным сопротивлениями определяется по закону Ома:

На векторной диаграмме видно, что в цепи переменного тока с активным и индуктивным сопротивлениями ток и напряжение не совпадают по фазе. Ток отстает от напряжения — на угол .

Угол сдвига между током и напряжением можно определить, если известен косинус этого угла.

Из треугольника напряжений косинус угла сдвига фаз

Теперь можно, пользуясь таблицей тригонометрических функ­ций, определить угол .

Пример. Падение напряжения на активном сопротивлении катушки Ua =30 в. Общее напряжение на ее зажимах U=60в. Определить угол сдвига фаз между током и напряжением в цепи.

Решение. На основании данных найдем

По таблице тригонометрических функций угол сдвига фаз при со =0,5 составляет 60°.

Цепь переменного тока с активным сопротивлением и индуктивностью — Студопедия

Рис.2.21 изображает неразветвлённую цепь с активным сопротивлением R и индуктивностью L.

Рис.2.21. Цепь переменного тока с активным сопротивлением и индуктивностью

Пусть мгновенный ток в цепи изменяется по закону . Тогда мгновенное напряжение на активном сопротивлении , так как на этом участке напряжение и ток совпадают по фазе. Напряжение на катушке индуктивности , поскольку на индуктивности напряжение опережает по фазе ток на угол .

Построим для действующих значений напряжения и тока векторную диаграмму для рассматриваемой цепи (рис. 2.22).

Векторы и образуют треугольник напряжений. Выведем закон Ома для этой цепи. Из треугольника напряжений имеем . Но , а , где — индуктивное сопротивление, следовательно:

, откуда

. (2.22)

Рис.2.22. Векторная диаграмма действующих значений тока и напряжения цепи переменного тока с активным сопротивлением и индуктивностью

Введем обозначение , где Z — полное сопротивление цепи. Тогда выражение закона Ома примет вид:

. (2.23)

Полное сопротивление Z можно определить из треугольника сопротивлений (рис. 2.23).

Рис.2.23. Треугольник сопротивлений цепи переменного тока с активным сопротивлением и индуктивностью


Сдвиг фаз между током и напряжением определяется из треугольника сопротивлений:

, (2.24)

. (2.25)

Поскольку вектор сдвинут по фазе относительно вектора на угол против часовой стрелки, этот угол имеет положительное значение.

Если , то мгновенная мощность . Для действующих значений произведение , откуда . Выражение . Исходя из этого,

. (2.26)

Таким образом, мгновенная мощность переменного тока может быть представлена в виде постоянной величины и, изменяющейся около неё с двойной частотой, величины (рис. 2.24).

Введем понятие средней или активной мощности:

. (2.27)

Активная мощность характеризует расход энергии на активном сопротивлении.

Реактивная мощность характеризует обмен энергий между индуктивной катушкой и источником:

. (2.28)

Полная мощность оценивает предельную мощность нагрузки:

. (2.29)

Рис.2.24. Зависимости мгновенных значений напряжения, тока и мощности цепи переменного тока с активным сопротивлением и индуктивностью

Совокупность всех мощностей можно определить из треугольника мощностей (рис. 2.25).

Рис.2.25. Треугольник мощностей

Так: Обозначим коэффициент мощности в виде соотношения .

Коэффициент мощности cosφ изменяется от 0 до 1. По его величине судят, какую часть полной мощности составляет активная мощность. На практике стремятся к увеличению cosφ.

4. Электрическая цепь с активным сопротивлением

Электрический ток проводимости в металлах представляет собой направленное движение свободных электронов. Скорость электронов и направление их движения определяется значением и полярностью приложенного к проводнику напряжения.

При движении электроны сталкиваются с атомами проводника, и их кинетическая энергия превращается в тепловую энергию, которая идет на нагрев проводника и рассеивание в окружающую среду. Этот активный процесс является необратимым и количественно определяется сопротивлением R. Поэтому его называют активным сопротивлением. Как правило, различают активное и омическое сопротивления. При рассмотрении цепей переменного тока сопротивление R называют активным, так как оно больше сопротивления постоянному току. При рассмотрении цепей постоянного тока сопротивление R называют омическим.

(Если через один и тот же проводник пропускать переменный и постоянный ток, то для того, чтобы выделилось одно и то же количество теплоты, величина переменного тока должна быть больше, чем постоянного).

Элементы электрической цепи, обладающие только активным сопротивлением R, называют резисторами. Все элементы электрической цепи – провода, обмотки, реостаты – обладают активным сопротивлением, все они нагреваются при пропускании тока.

Кроме активного сопротивления существует еще индуктивное, емкостное, реактивное, полное.

Пусть к зажимам цепи с активным сопротивлением R приложено напряжение источника питания.

По закону Ома для мгновенных значений для этой цепи

, отсюда

– закон Ома для амплитудных значений.

Разделив левую и правую части этого выражения на , получим

– закон Ома для действующих значений

( , – связь действующего значения с амплитудным).

– закон Ома в комплексной форме.

Комплексное действующее значение тока .

Поскольку ток и напряжение совпадают по фазе, как видно из формул для мгновенных значений, то можно представить изменение тока и напряжения на временной диаграмме следующим образом:

5. Мощность в цепи с активным сопротивлением

Мгновенная мощность

Мощность определяет скорость расхода энергии, и для цепей переменного тока также является переменной величиной.

Т.к. , то

Из формулы видно:мгновенная мощность остается всегда положительной и колеблется около уровня UI (см. график).

С энергетической точки зрения, положительная мощность – это мощность, которая поступает из генератора в нагрузку, а отрицательная – та, которая возвращается обратно. Первая мощность называется активной, вторая – реактивной. Через активное сопротивление проходит только активная мощность.

Средняя мощность

Для определения расхода энергии за длительное время пользуются средним значением мощности. Такую мощность еще называют активной

P = UI.

Активная мощность обозначается [P], единица измерения Ватт (Вт), (кВт), (МВт).

6. Электрическая цепь с индуктивностью

Индуктивность можно определить как меру магнитной инерции элемента в отношении электромагнитного поля. По смыслу индуктивность в электротехнике можно сравнить с массой в механике. Чем больше индуктивность элемента, тем медленнее и тем большую энергию магнитного поля он запасает.

Индуктивностью, индуктивным сопротивлением обладают в разной мере все элементы электрической цепи переменного тока. Особенно велика она у обмоток или катушек, состоящих из большого числа витков. В цепях постоянного тока индуктивность равна нулю, поскольку ток не меняется со временем и проявляется лишь при переходных режимах.

Рассмотрим электрическую цепь, состоящую из источника и катушки индуктивности.

Под действием синусоидального напряжения в цепи протекает синусоидальный ток

В результате этого вокруг катушки возникает переменное магнитное поле, и в катушке наводится э.д.с. самоиндукции.

Если R = 0, то напряжение источника целиком идет на уравновешивание этой э.д.с. Следовательно,

Т.к. по закону электромагнитной индукции , то

или , где .

Сопоставив выражения для мгновенных значений тока и напряжения, можно видеть, что напряжение в цепи с индуктивностью опережает по фазе ток на .

Т.О, временная диаграмма для тока и напряжения выглядит:

Векторная диаграмма тока и напряжения для цепи с индуктивностью

,

– индуктивное сопротивление цепи;

: видно, что с увеличением частоты увеличивается индуктивное сопротивление.

Закон Ома для цепи с индуктивностью

Закон Ома для амплитудных значений для цепи с индуктивностью: .

Разделив левую и правую части этого выражения на , получим закон Ома для действующих значений .

Закон Ома в комплексной форме

,

j обеспечивает сдвиг фаз на между I и U на комплексной плоскости.

Цепь с активным сопротивлением, индуктивностью и емкостью

⇐ ПредыдущаяСтр 17 из 33Следующая ⇒
 
 

Цепь с активным сопротивлением, индуктивностью и емкостью представляет собой общий случай последовательного соединения активных и реактивных сопротивлений и является последовательным колебательным контуром (рис. 5.12).

Рисунок 5.12 Схема цепи переменного тока с R, L и C и векторная диаграмма

 

Принимаем фазу тока нулевой: .

Тогда напряжение на активном сопротивлении , напряжение на индуктивности , напряжение на емкости . Построим векторную диаграмму при условии , т. е. .

Вектор результирующего напряжения U замыкает многоугольник векторов , и (рис. 5.12). Вектор определяет напряжение на индуктивности и емкости. Как видно из диаграммы, это напряжение может быть меньше напряжения на каждом из участков в отдельности. Это объясняется процессом обмена энергией между индуктивностью и емкостью. Выведем закон Ома для рассматриваемой цепи. Так как модуль вектора рассчитывают как разность действующих значений , то из диаграммы рис. 5.12 следует, что . Но , , ; следовательно, , откуда

(5.27)

Введя обозначение , где Z – полное сопротивление цепи, найдем

(5.28)

Разность между индуктивным и емкостным сопротивлениями называют реактивным сопротивлением цепи. Учитывая это, получим треугольник сопротивлений для цепи с R, L и С (рис. 5.13). При реактивное сопротивление положительно и сопротивление цепи носит активно-индуктивный характер. При реактивное сопротивление отрицательно и сопротивление цепи носит активно-емкостный характер. Знак сдвига фаз между током и напряжением получим автоматически, так как реактивное сопротивление — величина алгебраическая:

 
 

(5.29)

Рисунок 5.13 Треугольник сопротивлений для цепи с R, L и C

 

Таким образом, при преобладает или индуктивное, или емкостное сопротивление, т. е. с энергетической точки зрения цепь с R, L и С сводится к цепи с R, L или с R, С. Тогда мгновенная мощность

причем знак φ определяется по формуле (5.29). Соответственно активная, реактивная и полная мощности характеризуются выражениями ; ; .

 

©2015 arhivinfo.ru Все права принадлежат авторам размещенных материалов.

Последовательная цепь — переменный ток

Последовательная цепь — переменный ток

Cтраница 1

Последовательная цепь переменного тока имеет активное сопротивление 80 ом и емкостное 150 ом.  [1]

Последовательная цепь переменного тока с частотой 50 гц имеет активное сопротивление 12 ом, индуктивность 0 2 гн и емкость 50 мкф.  [2]

В последовательной цепи переменного тока, содержащей сопротивление R, емкость С и индуктивность L, между приложенным напряжением и током существует сдвиг по фазе.  [3]

В последовательной цепи квазистационарного переменного тока сила тока / в каждый момент времени во всех участках цепи одинакова.  [4]

Все рассмотренные разновидности последовательной цепи переменного тока могут быть характеризованы векторными диаграммами, приведенными в сводной таблице-диаграмме, показанной на рис. 9 — 37: а) цепь с активным сопротивлением; б) цепь с индуктивностью; в) цепь с емкостью; г) цепь с ак-тииным сопротивлением и индуктивностью; д) цепь с активным сопротивлением и емкостью; е) общий случай сложной цепи с преобладанием индуктивного сопротивления; ж) общий случай сложной цепи с преобладанием емкостного сопротивления.  [5]

Все рассмотренные разновидности последовательной цепи переменного тока могут быть характеризованы векторными диаграммами, приведенными в сводной таблице-диаграмме, показанной на рис. 9 — 37: а) цепь с активным сопротивлением; б) цель с индуктивностью; в) цепь с емкостью; г) цепь с активным сопротивлением и индуктивностью; д) цепь с активным сопротивлением и емкостью; е) общий случай сложной цепи с преобладанием индуктивного сопротивления; ж) общий случай сложной цепи с преобладанием емкостного сопротивления.  [6]

Эта формула выражает закон Ома для последовательной цепи переменного тока с индуктивностью и активным сопротивлением.  [7]

Рассмотрим, как изменяются ток и напряжение в последовательной цепи переменного тока.  [8]

Эта формула является общим выражением закона Ома для последовательной цепи переменного тока.  [10]

Это выражение является самым общим выражением закона Ома для последовательной цепи переменного тока.  [11]

Это выражение является самым общим выражением закона Ома для последовательной цепи переменного тока.  [12]

Необходимо подчеркнуть, что подобно тому, как в последовательной цепи переменного тока полное сопротивление цепи не равно сумме отдельных полных сопротивлений гц7 2г, так и в разветвленной цепи переменного тока эквивалентная полная проводимость не равна сумме полных проводимостей отдельных ветвей уа.  [13]

Рассмотрим, как изменяются сила тока и напряжение в последовательной цепи переменного тока. В цепи, содержащей все три вида сопротивления, ток i и напряжение и, оказываются сдвинутыми по фазе на некоторый угол ср ( рис. 68, б и б) больший 0 и меньший 90 в зависимости от отношения активного и реактивного сопротивлений цепи.  [14]

Установим при помощи топографической диаграммы еще одну важную закономерность для последовательной цепи переменного тока.  [15]

Страницы:      1    2

С АКТИВНЫМ, ИНДУКТИВНЫМ И ЕМКОСТНЫМ

СОПРОТИВЛЕНИЯМИ

 

При расчетах цепей переменного тока, так же как и цепей постоянного тока, используют законы Ома и Кирхгофа. Отличие в применении этих законов заключается в том, что в цепях переменного тока необходимо учитывать углы сдвига фаз между токами и напряжениями.

Цепь переменного тока, в которую включены последовательно активное сопротивление R, индуктивность L, обладающая индуктивным сопротивлением , и емкость С, имеющая емкостное сопротивление, изображена на рис. .8, а.

Под действием переменного напряжения и в этой цепи протекает переменный ток iСогласно закону Ома определим падения напряжения на каждом элементе цепи:

 

Эти падения напряжения имеют соответствующие углы сдвига фаз по отношению к общему току цепи I.

 

Построим векторную диаграмму тока и напряжений для рассматриваемой цепи (рис. .8,б). Так как сопротивления соединены последовательно, то в них протекает одинаковый ток. Отложим по горизонтали в выбранном масштабе вектор тока I. В цепи с активным сопротивлением ток и напряжение совпадают по фазе, поэтому вектор напряжения откладываем по вектору токаI . Напряжение на индуктивности опережает ток на угол = 90°. Поэтому вектор откладываем вверх под углом 90° к вектору тока I . В цепи семкостьюи наоборот, напряжение отстает от тока на угол = 90°, поэтому вектор откладываем на диаграмме вниз под углом 90° к вектору тока I.

 

Рис. .8. Цепь переменного тока с последовательно включенными сопротивлением ,индуктивностью и емкостью:

а — электрическая схема; б — векторная диаграмма; в — треугольник сопротивлений

 

Для определения общего напряжения, приложенного к зажимам

цепи, сложим векторы и. Для этого вычтем из большего вектора и

вектор и получим вектор ( ,) выражающий векторную сумму этих двух напряжений. Теперь сложим векторы ( ) .

Суммой этих векторов будет диагональ параллелограмма — вектор Uявляющийся общим напряжением на зажимах цепи:

 

2или 2

Полное сопротивление цепи переменного тока, содержащей активное, индуктивное и емкостное сопротивления,

 

2

Полная проводимость цепи:

 

Угол сдвига фаз между током и напряжением цепи определяется тригонометрическими функциями:

Если >, то вектор напряжения U опережает вектор тока I, а если < то Uотстает от тока I.Треугольник сопротивлений изображен на рис..8, в

 

Пример .4. В электрическую цепь, приведенную на рисунке, с напряжением U = 220 В последовательно включены: резистор с активным сопротивлением , = 5 Ом, индуктивная катушка Lс активным сопротивлением

= 3 Ом и индуктивным сопротивлением = 4 Ом, конденсатор С с емкостным сопротивлением = 10 Ом. Определить ток в цепи и напряжение на отдельных элементах цепи. Нарисовать векторную диаграмму.

Решение

Определяем общее сопротивление

 

2 = =10Ом

Ток в цепи

 

 

Напряжение на отдельных элементах цепи:

 

В

В

В

В

Векторная диаграмма приведена на рисунке ниже.

 

Рассмотрим электрическую цепь, состоящую из двух приемников, подключенных параллельно к зажимам источника синусоидального напряжения u= sin

(рис. .9,а). В первом приемнике включены последовательно элементы и L, во втором соответственно и С. Оба приемника находятся под действием одного общего напряженияu. Запишем выражения для мгновенных значений токов для первой и второй ветви:

 

Рис. 9. Цепь переменного тока с параллельно соединенными элементам

а — электрическая схема: б — векторная диаграмма

Так как в первую ветвь включена индуктивная катушка, а во вторую— конденсатор, то электрические цепи соответственно носят индуктивны й и емкостной характер.

Действующее значение тока I и косинус угла определяем из следующих выражений:

 

Значение тока I в неразветвленной части цепи определяют как геометрическую сумму токов в ветвях, т.е.I = +

Векторная диаграмма приведена на рис. 9, б. Токи, протекающие через ветви, состоят из активной и реактивной составляющих и соответственно имеют индуктивный и емкостной характер.

Ветвь с активной составляющей характеризуется активной проводимостью

 

Ветвь с реактивной составляющей характеризуется реактивной проводимостью . Для первой ветви реактивная проводимость

 

2

Реактивная проводимость не является величиной, обратной реактивному сопротивлению, так как при ее определении учитывается и активное сопротивлении ветви. Полная проводимость ветви

 

КОНТРОЛЬНЫЕ ВОПРОСЫ

1. Какие законы используют при расчете цепей переменного тока?

2.В чем состоит принципиальное отличие расчета цепей переменного

тока от расчета цепей постоянного тока?

 

 

РЕЗОНАНС НАПРЯЖЕНИЙ

В цепи переменного тока с активным, индуктивным и емкостным сопротивлениями, соединенными последовательно (рис. 10, а), может возникнуть резонанс напряжений

 

Резонанс напряжений наступает в том случае, если индуктивное сопротивление и емкостное сопротивление равны между собой, т.е.

 

Так как эти сопротивления зависят от частоты, резонанс наступает при определенной резонансной частоте :

 

 

При заданной частоте резонанс напряжений может быть достигнут изменением L и С.

 

Рис. 10. Цепь переменного тока при резонансе напряжения:

а — электрическая схема; б — векторная диаграмма

 

Когда цепь не настроена в резонанс, ее полное сопротивление

 

2

При резонансе напряжений в рассматриваемой цепи, когда = ее полное сопротивление

 

 

Запомните

1. Полное сопротивление цепи при резонансе напряжений равно активному сопротивлениюR.

2. Угол сдвига фаз между током и напряжением при резонансе равен нулю. Это значит, что ток и напряжение совпадают по фазе (как в цепи сактивным сопротивлением)

 

 

Уменьшение полного сопротивления цепи приводит к тому, что сила тока в ней возрастает. Напряжение генератора переменного тока, включенного в цепь, расходуется на активном сопротивлении:

 

Напряжение на индуктивности и емкости определяется согласно закону Ома. В связи стем что в последовательно соединенных сопротивлениях протекает одинаковый ток и при резонансе индуктивное сопротивление = напряжения на индуктивности и емкости тоже равны:

Векторная диаграмма для рассматриваемой цепи при резонансе напряжения приведена на рис 10, б. На векторной диаграмме видно, что напряжения на индуктивности и емкости равны, сдвинуты по фазе друг относительно друга на 180° и взаимно компенсируются.

Если одновременно увеличить оба реактивных сопротивления и , не нарушая при этом условия резонанса =, то соответственно возрастут оба частичных напряжения и, а сила тока в цепи при этом не изменится. Таким образом, можно получить и во много раз большие, чем напряжение U на зажимах цепи, поэтому резонанс напряжений — опасное явление для энергетических установок.

 

В электроустановках большой мощности резонанс напряжений, который может наступить внезапно, например при изменении емкостного сопротивления, может привести к опасным перенапряжениям и рассматривается как аварийный. В технике связи и автоматике явление резонанса напряжений широко используют для настройки приемных и передающих устройств на определенную частоту

 

Пример.5. В цепь переменного тока включены последовательно активное сопротивление R = 3 Ом, индуктивность L= 0,005 Гн и емкость С = 63,5 мкф. Генератор, включенный в цепь, вырабатывает переменное напряжение U = 2,5 В с резонансной частотой = 285 Гц. Определить индуктивное и емкостное сопротивления, полное сопротивление цепи, токпротекающий в цепи, напряжения на емкости и индуктивности.

 

 

Решение

Индуктивное сопротивление

 

Ом

Емкостное сопротивление

 

 

Индуктивное сопротивление равно емкостному, следовательно, в цепи наступает резонанс напряжения.

Полное сопротивление цепи при резонансе напряжения

 

2 = 2 = 3Ом

Сила тока в цепи

 

Напряжение на индуктивности

 

 

Напряжение на емкости

 

Как видно из представленного примера, в режиме резонанса напряжения на индуктивности и емкости равны и превышают напряжение генератора.

 

КОНТРОЛЬНЫЕ ВОПРОСЫ

1. Что называется резонансом напряжений?

2. Чему равно полное сопротивление цепи при резонансе напряжений?

3. Чему равен угол сдвига фаз между током и напряжением при резонансе напряжений?

4. Какую опасность представляет резонанс напряжений в технике?

 

 

РЕЗОНАНС ТОКОВ

В цепи переменного тока, в которой индуктивность L, емкость Си сопротивлениеR соединены параллельно (рис. .11, а), может возникнуть резонанс токов при условии равенства реактивных сопротивлений:

Ток при резонансе достигает минимального значения I = U/R, a cos = 1, т. е. достигает своего максимального значения.

 

Значение резонансной частоты определяется формулой

 

Из формулы следует, что, изменяя величину емкости или индуктивности контура, можно изменять (регулировать) частоту свободных колебаний, т. е. настраивать контур на определенную частоту.

Векторная диаграмма изображена на рис..11, б в режиме резонанса токов.

 

Рис .11. Цепь переменного тока при резонансе тока:

а-электрическая схема; б — векторная диаграмма

 

Свободные электрические колебания, возникающие в колебательном контуре, всегда затухающие. Затухание колебаний в контуре объясняется тем, что при прохождении электрического то в контуре энергия тратится на нагревание провода, из которого изготовлены индуктивная катушка и соединительные провода.

Потеря энергии в контуре вызывает постепенное уменьшение амплитуды свободных колебаний и их полное прекращение. Скорость затухания колебаний в контуре связана с потерей энергии в нем и зависит от сопротивления контура.

 

Запомните

■ Токи в ветвях, содержащих L и С, при резонансе тока могут быть

больше общего тока цепи.

■ Индуктивный и емкостной токи противоположны по фазе, равны по значению и по отношению к источнику энергии взаимно

компенсируются, т. е. идет обмен энергией между индуктивной

катушкой и конденсатором.

■ Реактивная мощность цепи при резонансе токов равна нулю, поскольку равны и противоположно направлены токи . Иными словами, реактивная мощность, потребляемая в индуктивной катушке, равна реактивной мощности, генерируемой в конденсаторе.

 

В результате резонанса токов общий ток в цепи может быть относительно мал, а в ветвях индуктивности и емкости, где происходят электрические колебания, переменный ток значительно больше общего.

 

 

Пример 6. Разветвленная цепь, приведенная на рисунке слева, имеет

следующие параметры: = = 10 Ом, R = 40 Ом, U= 120 В. Определить токи в ветвях и общий ток цепи. Построить векторную диаграмму.

Решение

Определим токи в ветвях:

 

Рассчитаем общий ток цепи

 

Векторная диаграмма изображена на рисунке справа. Общий ток цепи при резонансе токов в четыре раза меньше тока в ветвях, содержащих реактивные элементы.

КОНТРОЛЬНЫЕ ВОПРОСЫ

 

1. При каких условиях возникает резонанс токов?

2. От чего зависит скорость затухания колебаний в контуре?

3. Чему равна реактивная мощность цепи при резонансе токов?

 

Переменный ток в цепи с активным сопротивлением. Цепь переменного тока с активным сопротивлением

Для простоты пренебрегаем. К зажимам цепи приложено синусоидальное напряжение

u = U m × sin ωt .

В цепи, представленной на рисунке 1, приложенное внешнее напряжение уравновешивается в сопротивлении r , которое называется активным падением напряжения и обозначается U a .

U a = I × r .

Мгновенное значение мощности в рассматриваемой цепи равно произведению мгновенных значений напряжения и тока:

p = u × i .

На рисунке 3 дана кривая мгновенной мощности за один . Из чертежа видно, что мощность не является постоянной величиной, она пульсирует с двойной .


Рисунок 3. Кривая мгновенной мощности цепи с активным сопротивлением

Среднее за период значение мощности или просто средняя мощность обозначается буквой P и может быть определена по формуле, доказательство которой мы не приводим:

P = U × I × cos φ ,

где угол φ — угол между напряжением и током.

Средняя мощность называется также активной мощностью. Данная формула справедлива для любых цепей переменного тока.

Для цепи с активным сопротивлением напряжение и ток совпадают по фазе. Поэтому угол φ равен нулю, а cos φ = 1. Для активной мощности получим:

P = U × I

P = I 2 × r ,

то есть формула мощности для цепи переменного тока с активным сопротивлением такая же, как формула мощности для цепи постоянного тока. Активным сопротивлением обладают все проводники. В цепи переменного тока практически только одним активным сопротивлением обладают нити ламп накаливания, спирали электронагревательных приборов и реостатов, дуговые лампы, специальные бифилярные обмотки и прямолинейные проводники небольшой длины.

1 Пульсацией называется изменение численного значения величины при постоянстве ее знака.

Рассмотрим цепь (фиг. 140), состоящую из сопротивления г. Влиянием индуктивности и емкости для простоты пренебрегаем.

К зажимам цепи приложено синусоидальное напряжение

Как следует из последнего выражения, вид закона Ома для цепи переменного тока, содержащей сопротивление, тот же, что для цепи постоянного тока. Кроме того, из закона Ома вид-на пропорциональность между мгновенным значением напряжения и мгновенным значением тока. Отсюда следует, что в цепи переменного тока, содержащей сопротивление г, напряжение и ток совпадают по фазе. На фиг. 141 даны кривые напряжения и тока и векторная диаграмма для рассматриваемой цепи, причем длины векторов обозначают действующие значения напряжения и тока. Сопротивление проводников переменному току несколько больше их сопротивления постоянному току. Это объясняется поверхностным эффектом, сущность которого изложена в 87. Поэтому сопротивление проводников переменному току называют активным. Обозначается оно также буквой r.

В цепи, представленной на фиг. 140, приложенное внешнее напряжение уравновешивается падением напряжения в сопротивлении r, которое называется активным падением напряжения и обозначается U a

Мгновенное значение мощности в рассматриваемой цепи равно произведению мгновенных значений напряжения и тока:

На фиг. 142 дана кривая мгновенной мощности за один период. Из чертежа видно, что мощность не является постоянной величиной, она пульсирует с двойной частотой.

Среднее за период значение мощности или просто средняя мощность обозначается буквой Р и может быть определена по формуле, доказательство которой мы не приводим:

Где — угол сдвига фаз между напряжением и током.

Средняя мощность называется также активной мощностью. Данная формула активной мощности справедлива для любых цепей переменного тока.

Для цепи с активным сопротивлением напряжение и ток совпадают по фазе. Поэтому угол равен нулю, a cos=1. Для активной мощности получим:

Т. е. формула мощности для цепи переменного тока с активным сопротивлением такая же, как формула мощности для цепи постоянного тока. Активным сопротивлением обладают все проводники. В цепи переменного тока практически только одним активным сопротивлением обладают нити ламп накаливания, спирали электронагревательных приборов и реостатов, дуговые лампы, специальные бифилярные обмотки и прямолинейные проводники небольшой длины.

Если цепь переменного тока содержит только резистор R (лампа накаливания, электронагревательный прибор и т. д.), к которому приложено переменное синусоидальное напряжение и (рис. 1-5, а):

то ток i в цепи будет определяться значением этого сопротивления:

где — амплитуда тока; при этом ток i и напряжение и совпадают по фазе. Обе эти величины, как видно, можно изобразить на временной (рис. 1-5, б) и векторной (1-5, в) диаграммах. Теперь установим, как изменяется мощность в любой момент времени — мгновенная мощность, характеризующая собой скорость преобразования электрической энергии в другие виды энергии в данный момент времени

где IU — произведение действующих значений тока и напряжения.

Из полученного следует, что мощность в течение периода остается положительной и пульсирует с удвоенной частотой. Графически это можно представить так, как показано на рисунке 1-6. В этом случае электрическая энергия превращается необратимо, например, в теплоту независимо от направления тока в цепи.

Кроме мгновенного значения мощности различают еще среднюю мощность за период:

но так как второй интеграл равен нулю, то окончательно имеем:

Средняя за период мощность переменного тока называется активной мощностью, а соответствующее ей сопротивление — активным.

Средняя мощность и активное сопротивление связаны с безвозвратным преобразованием электрической энергии в другие виды энергии. Активное сопротивление электрической цепи не сводится только к

сопротивлению проводников, в которых электрическая энергия превращается в теплоту. Это понятие значительно шире, так как средняя мощность электрической цепи равна сумме мощностей всех видов энергии, полученной из электрической, на всех участках цепи (теплота, механическая и др.).

Из полученных соотношений следует, что

которое является математической записью закона Ома для цепи переменного тока с активным сопротивлением.

Вынужденные Электромагнитные Колебания

Вынужденными электромагнитными колебаниями называют периодические изменения силы тока и напряжения в электрической цепи, происходящие под действием переменной ЭДС от внешнего источника. Внешним источником ЭДС в электрических цепях являются генераторы переменного тока, работающие на электростанциях.

Принцип действия генератора переменного тока легко показать при рассмотрении вращающейся рамки провода в магнитном поле.

В однородное магнитное поле с индукцией В помещаем прямоугольную рамку, образованную проводниками (abсd).

Пусть плоскость рамки перпендикулярна индукции магнитного поля В и ее площадь равна S.

Магнитный поток в момент времени t 0 = 0 будет равен Ф = В*8.

При равномерном вращении рамки вокруг оси OO 1 с угловой скоростью w магнитный поток, пронизывающий рамку, будет изменяться с течением времени по закону:

Изменение магнитного потока возбуждает в рамке ЭДС индукцию, равную

где Е 0 = ВSw — амплитуда ЭДС.

Если с помощью контактных колец и скользящих по ним щеток соединить концы рамки с электрической цепью, то под действием ЭДС индукции, изменяющейся со временем по гармоническому закону, в электрической цепи возникнут вынужденные гармонические колебания силы тока — переменный ток .

На практике синусоидальная ЭДС возбуждается не путем вращения рамки в магнитном поле, а путем вращения магнита или электромагнита (ротора) внутри статора — неподвижных обмоток, навитых на сердечники из магнитомягкого материала. В этих обмотках находится переменная ЭДС, что позволяет избежать снятия напряжения с помощью контактных колец.

Переменный Ток

Рассмотрим процессы, происходящие в проводнике, включенном в цепь переменного тока.

Если индуктивность проводника настолько мала, что при включении его в цепь переменного тока индукционными полями можно пренебречь по сравнению с внешним электрическим полем, то движение электрических зарядов в проводнике определяется действием только внешнего электрического поля, напряженность которого пропорциональна напряжению на концах проводника.

При изменении напряжения по гармоническому закону U = U m cos wt напряженность электрического поля в проводнике изменяется по такому же закону.

Под действием переменного электрического поля в проводнике возникает переменный электрический ток, частота и фаза колебаний которого совпадает с частотой и фазой колебаний напряжения:

где i — мгновенное значение силы тока, I m — амплитудное значение силы тока.

Колебания силы тока в цепи являются вынужденными электрическими колебаниями, возникающими под действием приложенного переменного напряжения.

Амплитуда силы тока равна:

При совпадении фаз колебаний силы тока и напряжения мгновенная мощность переменного тока равна:

Среднее значение квадрата косинуса за период равно 0,5. В результате средняя мощность за период

Для того чтобы формула для расчета мощности переменного тока совпадала по форме с аналогичной формулой для постоянного тока (Р = PR), вводится понятие действующих значений силы тока и напряжения. Из равенства мощностей получим

Действующим значением силы тока называют величину, в корень из 2 раз меньшую ее амплитудного значения:

Действующее значение силы тока равно силе такого постоянного тока, при котором средняя мощность, выделяющаяся в проводнике в цепи переменного тока, равна мощности, выделяющейся в том же проводнике в цепи постоянного тока.

Действующее значение переменного напряжения в корень из 2 раз меньше его амплитудного значения:

Средняя мощность переменного тока при совпадении фаз колебаний силы тока и напряжения равна произведению действующих значений силы тока и напряжения:

Сопротивление элемента электрической цепи, в которой происходит превращение электрической энергии во внутреннюю энергию, называют активным сопротивлением . Активное сопротивление участка цепи можно определить как частное от деления средней мощности на квадрат действующего значения силы тока:

Активным сопротивлением R называется физическая величина, равная отношению мощности к квадрату силы тока , что получается из выражения для мощности . При небольших частотах практически не зависит от частоты и совпадает с электрическим сопротивлением проводника.

Пусть в цепь переменного тока включена катушка. Тогда при изменении силы тока по закону в катушке возникает ЭДС самоиндукции . Т.к. электрическое сопротивление катушки равно нулю, то ЭДС равна минус напряжению на концах катушки, созданному внешним генератором (??? Каким еще генератором???) . Следовательно, изменение силы тока вызывает изменение напряжения, но со сдвигом по фазе . Произведение является амплитудой колебаний напряжение, т.е. . Отношение амплитуды колебаний напряжения на катушке к амплитуде колебаний тока называется индуктивным сопротивлением .

Пусть в цепи находится конденсатор. При его включение он четверть периода заряжается, потом столько же разряжается, потом то же самое, но со сменой полярности. При изменении напряжения на конденсаторе по гармоническому закону заряд на его обкладках равен . Ток в цепи возникает при изменении заряда: , аналогично случаю с катушкой амплитуда колебаний силы тока равна . Величина, равная отношению амплитуды к силе тока, называется емкостным сопротивлением .

АКТИВНОЕ СОПРОТИВЛЕНИЕ. ДЕЙСТВУЮЩИЕ ЗНАЧЕНИЯ СИЛЫ ТОКА И НАПРЯЖЕНИЯ Перейдем к более детальному рассмотрению процессов, которые происходят в цепи, подключенной к источнику переменного напряжения.

Сила тока в цени с резистором. Пусть цепь состоит из соединительных проводов и нагрузки с малой индуктивностью и большим сопротивлением R (рис. 4.10). Эту величину, которую мы до сих пор называли электрическим сопротивлением или просто сопротивлением, теперь будем называть активным сопротивлением. Сопротивление R называется активным, потому что при наличии нагрузки, обладающей этим сопротивлением, цепь поглощает энергию, поступающую от генератора. Эта энергия превращается во внутреннюю энергию проводников — они нагреваются. Будем считать, что напряжение на зажимах цепи меняется по гармоническому закону: u = U m cos t. Как и в случае постоянного тока, мгновенное значение силы тока прямо пропорционально мгновенному значению напряжения. Поэтому для нахождения мгновенного значения силы тока можно применить закон Ома : В проводнике с активным сопротивлением колебания силы тока совпадают по фазе с колебаниями напряже ния (рис. 4.11), а амплитуда силы тока определяется равенством

Мощность в цепи с резистором. В цепи переменного тока промышленной частоты (v = 50 Гц) сила тока и напряжение изменяются сравнительно быстро. Поэтому при прохождении тока по проводнику, например по нити электрической лампочки, количество выделенной энергии также будет быстро меняться со временем. Но этих быстрых изменений мы не замечаем.

Как правило, нам нужно бывает знать среднюю мощ ностъ тока на участке цепи за большой промежуток времени, включающий много периодов. Для этого достаточно найчи среднюю мощность за один период. Под средней за период, мощностью переменного тока понимают отношение суммарной энергии , поступающей в цепь за период, к периоду.

Мощность в цепи постоянного тока на участке с сопротивлением R определяется формулой P = I 2 R. (4.18) На протяжении очень малого интервала времени переменный ток можно считать практически постоянным. Поэтому мгновенная моoность в цепи переменного тока на участке, имеющем активное сопротивление R, определяется формулой P = i 2 R. (4.19) Найдем среднее значение мощности за период. Для этого сначала преобразуем формулу (4.19), подставляя в нее выражение (4.16) для силы тока и используя известное из математики соотношение График зависимости мгновенной мощности от времени изображен на рисунке 4.12, а. Согласно графику (рис. 4.12, б.), на протяжении одной восьмой периода, когда , мощность в любой момент времени больше, чем . Зато на протяжении следующей восьмой части периода, когда cos 2t Действующие значения силы тока и напряжения . Из формулы (4.21) видно, что величина есть среднее за период значение квадрата силы тока:

Величина, равная квадратному корню из среднего значения квадрата силы тока, называется действующим значением силы неременного тока. Действующее зртачепие силы неременного тока обозначается через I: Действующее значение силы переменного тока равно силе такого постоянного тока, при котором в проводнике выделяется то же количество теплоты , что и при переменном токе за то же время.

Действующее значение переменного напряжения определяется аналогично действующему значению силы тока: Заменяя в формуле (4.17) амплитудные значения силы тока и напряжения на их действующие значения, получаем Это закон Ома для участка цепи переменного тока с резистором.

Как и при механических колебаниях, в случае электрических колебаний обычно нас не интересуют значения силы тока, напряжения и других величин в каждый момент времени. Важны общие характеристики колебаний, такие, как амплитуда, период, частота, действующие значения силы тока и напряжения, средняя мощность. Именно действующие значения силы тока и напряжения регистрируют амперметры ивольтметры переменного тока.

Кроме того, действующие значения удобнее мгновенных значений еще и потому, что именно они непосредственно определяют среднее значение мощности Р переменного тока: P = I 2 R = UI. Колебания силы тока в цепи с резистором совпадают по фазе с колебаниями напряжения, а мощность определяется действующими значениями силы тока и напряжения.

Переменный ток оценивается его действием, эквивалентной действия постоянного тока. Активным сопротивлением называют такое сопротивление проводника, в котором электрическая энергия необратимо превращается во внутреннюю. Пусть напряжение в цепи переменного тока изменяется по гармоничным законом. Под действием переменного электрического поля в проводнике возникает переменный ток, частота и фаза колебаний которого совпадает с частотой и фазой колебания напряжения. Амплитудное значение силы тока равна отношению амплитудного значения напряжения к сопротивлению проводника. Мощность тока равна произведению силы тока и напряжения. Тогда активное сопротивление можно определить как отношение мощности переменного тока на участке цепи к квадрату действующей силы тока. Действующим значением силы тока называется сила постоянного тока, благодаря которой в проводнике выделяется за одинаковое время такое же количество теплоты, что и переменным током. Найти действующее значение силы тока можно как отношение амплитудного значения силы тока до квадратного корня из двух. Действующее значение напряжения также в корень из двух меньше его амплитудного значения.

При изучении вынужденных механических колебаний мы ознакомились с явлением резонанса . Резонанс наблюдается в том случае, когда собственная частота колебаний системы совпадает с частотой изменения внешней силы. Если трение мало, то амплитуда установившихся вынужденных колебаний при резонансе резко увеличивается. Совпадение вида уравнений для описания механических и электромагнитных колебаний (позволяет сделать заключение о возможности резонанса также и в электрической цепи, если эта цепь представляет собой колебательный контур, обладающий определенной собственной частотой колебаний.

При механических колебаниях резонанс выражен отчетливо при малых значениях коэфициента трения . В электрической цепи роль коэффициента трения выполняет ее активное сопротивление R. Ведь именно наличие этого сопротивления в цепи приводит к превращению энергии тока но внутреннюю энергию проводника (проводник нагревается). Поэтому резонанс в электрическом колебательном кон-lype должен быть выражен отчетливо при малом активном сопротивлении R.

Мы с вами уже знаем, что если активное сопротивление мало, то собственная циклическая частота колебаний в контуре определяется формулой Сила тока при вынужденных колебаниях должна достигать максимальных значений, когда частота переменного напряжения, приложенного к контуру, равна собственной частоте колебательного контура: Резонансом в электрическом колебательном контуре называется явление резкого возрастания амплитуды вынужденных колебаний силы тока при совпадении частоты внегннего переменного напряжения с собственной частотой колебательного контура.

Амплитуда силы тока при резонансе. Как и в случае механического резонанса, при резонансе в колебательном контуре создаются оптимальные условия для поступления энергии от внешнего источника в контур. Мощность в контуре максимальна в том случае, когда сила тока совпадает по фазе с напряжением. Здесь наблюдается полная аналогия с механическими колебаниями: при резонансе в механической колебательной системе внешняя сила (аналог напряжения в цепи) совпадает по фазе со скоростью (аналог силы тока).

Не сразу после включения внешнего переменного напряжения в цепи устанавливается резонансное значение силы тока. Амплитуда колебаний силы тока нарастает постепенно — до тех пор, пока энергия, выделяющаяся за период на резисторе, не сравняется с энергией , поступающей в контур за это же время:

Отсюда амплитуда установившихся колебаний силы тока при резонансе определяется уравнением При R 0 резонансное значение силы тока неограниченно возрастает: (I m) рез . Наоборот, с увеличением R максимальное значение силы тока уменьшается, и при больших R говорить о резонансе уже не имеет смысла. Зависимость амплитуды силы тока от частоты при различных сопротивлениях (R 1 Одновременно с увеличением силы тока при резонансе резко возрастают напряжения на конденсаторе и катушке индуктивности. Эти напряжения при ма.пом активном сопротивлении во много раз превышают внешнее напряжение.

Использование резонанса в радиосвязи. Явление электрического резонанса широко используется при осуществлении радиосвязи. Радиоволны от различных передающих станций возбуждают в антенне радиоприемника переменные токи различных частот, так как каждая передающая радиостанция работает на своей частоте. С антенной индуктивно связан колебательный контур (рис. 4.20). Вследствие электромагнитной индукции в контурной катушке возникают переменные ЭДС соответствующих частот и вынужденные колебания силы тока тех же частот. Но только при резонансе колебания силы тока в контуре и напряжения в нем будут значительными, т. е. из колебаний различных частот, возбуждаемых в антенне, контур выделяет только те, частота которых равна его собственной частоте. Настройка контура на нужную частоту обычно осуществляется путем изменения емкости конденсатора . В этом обычно состоит настройка радиоприемника на определенную радиостанцию. Необходимость учета возможности резонанса в электрической цепи. В некоторых случаях резонанс в электрической цепи может принести большой вред. Если цепь не рассчитана на работу в условиях резонанса, то его возникновеие может привести к аварии.

Чрезмерно большие токи могут перегреть провода. Большие напряжения приводят к пробою изоляции.

Такого рода аварии нередко случались еще сравнительно недавно, когда плохо представляли себе законы электрических колебаний и не умели правильно рассчитывать электрические цепи .

При вынужденных электромагнитных колебаниях возможен резонанс — резкое возрастание амплитуды колебаний силы тока и напряжения при совпадении частоты внешнего переменного напряжения с собственной частотой колебаний. На явлении резонанса основана вся радиосвязь.

Изучение цепей переменного тока с активным, емкостным и индуктивным сопротивлениями происходит в следующей логической последовательности: сначала дается понятие о том или ином виде сопротивлений в цепи переменного тока (сравнение с его поведением в цепи постоянного тока), затем фазовые соотношения, формула соответствующего сопротивления, преобразования энергии в цепи, содержащей только активное, емкостное или индуктивное сопротивление. Последовательность изучения сопротивлений в цепи переменного тока может быть и несколько иной. Понятие действующего значения силы тока и напряжения можно ввести так: вначале выводят выражение для расчета мгновенных значений мощности на активном сопротивлении, отсюда находят среднее значение мощности за период и выясняют, что есть среднее значение квадрата силы тока за период. Вводят определение: корень квадратный из этой величины называют действующим значением переменного тока. Название связано с тем, что при прохождении такого тока по участку с активным сопротивлением выделяется мощность

Такая же мощность выделяется в цепи постоянного тока, величина которого равна действующему значению переменного тока. Итак, действующим значением переменного тока является такое значение постоянного тока, которое в резисторе R выделяет такое же количество теплоты, что и переменный ток. Очень важно отметить, что шкалы электроизмерительных приборов, для измерения переменных с

илы тока и напряжения градуируют именно в действующих значениях этих величин. Рассмотрение цепи переменного тока со смешанным сопротивлением начинают с эксперимента — измеряют напряжение на каждом из последовательно включенных элементов цепи (лампе, катушке и батарее конденсаторов), подключенных к источнику переменного напряжения. Обращают внимание на следующие опытные факты: 1. Общее напряжение не равно сумме напряжений на отдельных участках, как это имело место для цепей постоянного тока. 2. Напряжение на участке, включающем в себя катушку и конденсатор, равно не сумме, а разности напряжений на каждом из них в отдельности. Объяснить этот результат можно предложить самим учащимся; им известно, что на индуктивности напряжение опережает ток на π/2, а на электроемкости отстает от него на ту же величину. Так как мгновенное значение силы тока в цепи всюду одно и то же, то ясно, что колебания напряжения на индуктивности и электроемкости происходят со сдвигом фаз, равным π, т. е. их фазы противоположны. 3. Полное сопротивление цепи меньше суммы всех включенных в нее сопротивлений (активного, индуктивного и емкостного). Учащихся нужно убедить, что чем меньше сдвиг фаз между током и напряжением, тем большую часть мощности, подводимой к цепи, используют полезно, необратимо превращая в другие виды энергии. Далее рассматривают устройство и работу трансформатора. На примере однофазного трансформатора показывают его действие (повышение и понижение напряжения) и устройство. Вначале рассматривают режим холостого хода, а затем нагруженного трансформатора. В качестве нагрузки целесообразно использовать реостат, так как им проще изменять нагрузку. Показывают, что при увеличении нагрузки возрастает сила тока как во вторичной, так и в первичной обмотке трансформатора. Учащимся предлагают самим с энергетических позиций объяснить возрастание силы тока в первичной цепи (увеличение потребления энергии на нагрузке естественно должно сопровождаться увеличением потребления энергии первичной обмоткой от генератора). Для изучения электромагнитных колебаний широко используется школьный прибор- звуковой генератор школьный ГЗШ. Он перекрывает диапазон генерируемых частот синусоидальных колебаний от 20 до 20000 Гц с диапазонами: «X1» (от 20 до 200 Гц), «X10» (от 200 до 2000 Гц), «X100» (от 2000 до 20000 Гц), питается от сети переменного тока напряжением 220 В. На лицевую панель генератора выведены тумблёр включения генератора в сеть, сигнальная лампочка, переключатель поддиапазонов на три фиксированных положения, отмеченных «X1», «X10», «X100», диск с неравномерной шкалой деления (от 20 до 200) ручка переменного резистора, позволяющая менять амплитуду выходного сигнала, выходные зажимы, рассчитанные на подключение цепей с разным сопротивлением (5, 600, 5000 Ом). Если для опытов необходимы частоты 20 – 200 Гц, то переключатель устанавливают в положение «X1» если 200 – 2000 Гц – в положение «X10», а для частот 2000 – 20000 Гц используют положение «X100». Плавную регулировку частоты осуществляют поворотом диска. Так же широко используются выпрямители ВУП-1 и ВУП-2 ВУП-2 предназначен для обеспечения питанием демонстрационных установок в опытах по электричеству. Технические данные: Прибор позволяет получить на выходных зажимах: выпрямленное напряжение 350В при максимальной силе тока 220мА; постоянное отфильтрованное напряжение 250В при максимальной нагрузке 50мА; регулируемое напряжение от 0 до 250В постоянного тока до 50мА; регулируемое напряжение от 0 до +100В и от 0 до-100В постоянного тока до 10мА; напряжение 6.3В переменного тока до 3А. Еще один источник питания без которого практически невозможно осуществлять многие опыты по электричеству РНШ. Регулятор напряжения школьный предназначен для плавного регулирования напряжения однофазного переменного тока с частотой 50 Гц, при проведении лабораторных и демонстрационных опытов в физических кабинетах школ. Прибор присоединяется к сети выводным шнуром. Прибор можно включить в сеть напряжением 127 и 220В. Рабочее напряжение снимается с зажимов, обозначенных «Выходное напряжение». В целях правильной эксплуатации регулятора напряжения в паспорте прибора приведена таблица допустимых значений электрической мощности нагрузки регулятора при разных напряжениях, подаваемых на нагрузку, и при сетевых напряжениях 127 и 220В. Установленный в регуляторе напряжения вольтметр имеет неравномерную шкалу. Достоверный отсчет можно вести только при 50В. В случае необходимости снимать с регулятора более низкие напряжения нужно параллельно выходным зажимам подключать дополнительный вольтметр с соответствующим пределом измерения. Регулятор напряжения может быть использован как для повышения, так и для понижения напряжений переменного тока, при разных демонстрационных и лабораторных опытах Для наглядного отображения электромагнитных колебаний применяют школьные осциллографы ОДШ-2 и ОЭШ-70. Наиболее широко при­меняют осциллографы для исследования периодических процессов, а также для изучения вольтамперных характеристик диода и трио­да, петли гистерезиса и др. В простейшем случае осциллограф состоит из четырех блоков: блока электронно-лучевой трубки ЭЛТ, генератора развертки ГР, усилителя исследуемого сигнала УС и блока питания БП. Основным элементом первого блока является электронно-лучевая трубка, на экране которой формируется картина исследуемого сиг­нала (осциллограмма). Нить накала НН подогревает ка­тод К, с поверхности которого вылетают элект­роны. Электроны, пролетев через отверстия уп­равляющего электрода, фокусирующего ци­линдра ФЦ и анода А, а также между пласти­нами XX и УУ, попадают на экран и вызывают его свечение. Изменением разности потенциалов между катодом и управляющим электродом мож­но менять число электронов в пучке, а это по­зволяет регулировать яркость изображения на экране. Чем больше по модулю отрицательный потенциал на управляющем электроде относи­тельно катода, тем меньше электронов пройдет через управляющий электрод и достигнет анода. Осциллограф снабжен ручкой «яркость» для уп­равления потоком электронов в пучке. Электрическое поле между фокусирующим цилиндром и анодом способно фокусировать рас­ходящийся электронный пучок. Обычно на передней стенке смонтированы выключатель сети, сигнальная лампочка, за­жимы «Вход У», «Вход X» и делитель входного сигнала. На боко­вую панель выведены ручки управления электронным пучком, «Синхронизация», «Внутр. — от сети — внешн.», «Уси­ление», ручки развертки, «Диапазоны 0, 30, 150, 500 Гц, 2, 8, 16 кГц», «Частота плавно», а также ручки усиления сигнала «Уси­ление У», «Усиление X». Осциллограф ОДШ-2 отличается от ОЭШ-70 конструктивно и внешним оформлением. На переднюю панель выведен не только экран электронно-лучевой трубки, но и основные ручки уп­равления. Верхний ряд ручек предназначен для управления элек­тронным пучком: «Яркость», «Фокус», «Вверх-вниз», «Влево-вправо». Во втором ряду сверху смонтированы ручки управления уси­лителем «Усиление У» и делитель напряжения 1:1, 1:10, 1:30, 1:1OO, 1:1000, а также выключатель сети с сигнальной лампоч­кой. В третьем ряду сверху расположены ручки и кнопки генера­тора развертки: «Частота плавно», «Вкл. 1, 2, 3, 4», «Усиление X». Кнопочный переключатель позволяет менять пилообразное напря­жение частотой от 20 Гц до 20 кГц. Генератор развертки рабо­тает только при нажатой кнопке «Вкл». В нижнем ряду располо­жены зажимы «Вход У», «Вход X», «Внешн. синхр», кнопки син­хронизации «Внешн.», «Внутр.» и ручка синхронизации. На боковую панель осциллографа ОДШ-2 выведены ручки уп­равления двухканальным коммутатором с двумя входами. Комму­татор позволяет наблюдать на экране осциллографа одновремен­но сигналы от двух источников переменного тока. Если частоты источников одинаковы, то по полученным осциллограммам мож­но судить о сдвиге фаз поданных сигналов. Например, на один вход можно подать сигнал, пропорциональный напряжению на конденсаторе, а на другой — пропорциональный силе тока, текущего через конденсатор. Тогда на экране осциллографа можно наблю­дать две синусоиды, сдвинутые по фазе на 90°. Применяя комму­татор, можно сравнивать частоту исследуемого сигнала со стан­дартной частотой, если эти сигналы отличаются по частоте. На задней стенке осциллографов ОДШ-2 и ОЭШ-70 смонтиро­ваны гнезда, позволяющие подавать исследуемый сигнал непосред­ственно на пластины электронно-лучевой трубки. Возможность подавать исследуемый сигнал непосредственно на пластины позво­ляет применить осциллограф и для цепей постоянного тока. По­давая сигнал постоянного напряжения на пластины XX (или УУ) при отключенной развертке, можно наблюдать смещение светя­щейся точки по горизонтали (или вертикали), причем отклонение этой точки пропорционально приложенному напряжению. Следо­вательно, осциллограф можно применить как вольтметр с боль­шим внутренним сопротивлением. Для усиления электромагнитных колебаний применяют усилители низкой частоты. Усилитель низкой частоты– электронный прибор. Предназначенный для усиления электрических колебаний звуковой частоты от 20 Гц до 20 кГц. Обычно усилитель состоит из нескольких блоков: предварительного усилителя напряжения, усилителя мощности, согласующего выходного трансформатора и блока питания. Для школ выпускаются усилители разной конструкции и отличающиеся по внешнему виду. Усилитель УНЧ-3на лицевой панели имеет ручку регулятора громкости и сигнальную лампочку. Ручкой регулятора громкости производят также включение и выключение сети. В крайнем левом положении ручки при повороте против часовой стрелки прибор отключен. Включение осуществляют поворотом ручки по часовой стрелке после щелчка. Так как усилитель собран на электронных лампах, то он начинает работать после их прогрева. На боковой стенке смонтированы три входных гнезда: для подключения М – микрофона, АД – адаптера, Л – линии. Нижние гнёзда соединены с корпусом прибора. На задней стенке имеются две пары гнёзд: Гр – для подключения громкоговорителя (низкоомный выход) и Л – высокоомный выход. Здесь же имеются вывод сетевого шнура с вилкой и октальная панель, в которую вставлена специальная вилка с предохранителем (на 0,5 А) для сети с напряжением 220 В. Вилку можно устанавливать в двух положениях: «220 В» и «127 В». Усилитель УНЧ-5 собран на транзисторах. На лицевой панели усилителя смонтирован выключатель сети с индикаторной лампочкой, гнезда выхода, гнезда входа для микрофона и звукоснимателя, разъем для подключения микрофона, ручки регулировки тембра по низкой и высокой частоте, ручка регулировки уровня сигнала, индикатор перегрузки. На задней стенке имеются вывод сетевого шнура с вилкой и предохранитель (на 0,5 А). На вход усилителя могут подаваться сигналы не только с микрофона и звукоснимателя, но и от других датчиков электрических колебаний напряжением от нескольких милливольт до вольт (сигналы с элементов цепи переменного тока, звукового генератора и т.д.). К выходу усилителя можно подключить не только громкоговоритель, но и другие приборы: осциллограф, измерительные приборы переменного тока, головные телефоны и пр. Потребляемая усилителем мощность не более 40 Вт, выходная – около 5 Вт. Запрещается при эксплуатации усилителя менять предохранитель, разбирать и ремонтировать прибор, включенный в сеть. Усилитель на вертикальной панели входит в комплект демонстрационных приборов по радиотехнике. Слева смонтированы универсальные зажимы входа усилителя. Первая лампа работает в режиме усиления напряжения, вторая – как усилитель мощности. В анодную цепь второй лампы включен согласующий трансформатор, вторичная обмотка которого соединена с зажимами низкого и высокого выходного напряжения. Три нижних зажима служат для подключения питания от ВУП –2, на два нижних зажима подаётся напряжение переменного тока 6,3 В для питания накала ламп, а на средний и третий снизу – напряжение постоянного тока 250 В для анодной цепи ламп, причём на третий снизу зажим подаётся положительный потенциал. Подключение блока питания и сборку установок с усилителем на панели запрещается выполнять при включённом в сеть выпрямителе ВУП-2. В демонстрационных установках предпочтение следует отдавать усилителю УНЧ-5.

Цепи индуктивности переменного тока

| Реактивное сопротивление и импеданс — индуктивный

Резисторы и индукторы

Катушки индуктивности ведут себя иначе, чем резисторы. В то время как резисторы просто препятствуют прохождению тока через них (снижая напряжение, прямо пропорциональное току), индукторы противодействуют изменениям тока через них, понижая напряжение, прямо пропорциональное скорости изменения тока .

В соответствии с законом Ленца , это индуцированное напряжение всегда имеет такую ​​полярность, чтобы поддерживать ток на его текущем значении.То есть, если ток увеличивается по величине, индуцированное напряжение будет «противодействовать» протеканию тока; если ток уменьшается, полярность изменится на противоположную и «подтолкнет» ток, чтобы противодействовать уменьшению.

Это противодействие текущему изменению называется реактивным сопротивлением , а не сопротивлением. Выражаясь математически, соотношение между падением напряжения на катушке индуктивности и скоростью изменения тока через катушку индуктивности выглядит следующим образом:

Переменный ток в простой индуктивной цепи

Выражение di / dt получено из расчетов, означающее скорость изменения мгновенного тока (i) во времени в амперах в секунду.

Индуктивность (L) измеряется в Генри, а мгновенное напряжение (е), конечно, выражается в вольтах. Иногда вы можете встретить скорость мгновенного напряжения, выраженную как «v» вместо «e» (v = L di / dt), но это означает то же самое.

Чтобы показать, что происходит с переменным током, давайте проанализируем простую цепь индуктивности:

Чистая индуктивная цепь: ток индуктора отстает от напряжения индуктора на 90 °.

Если бы мы изобразили ток и напряжение для этой очень простой схемы, это выглядело бы примерно так:

Чистая индуктивная цепь, формы сигналов.

Помните, что падение напряжения на катушке индуктивности является реакцией на изменение тока через нее.

Следовательно, мгновенное напряжение равно нулю всякий раз, когда мгновенный ток находится на пике (нулевое изменение или наклон уровня на синусоидальной волне тока), а мгновенное напряжение находится на пике везде, где мгновенный ток имеет максимальное изменение (точки крутизны на текущей волне, где она пересекает нулевую линию).

Это приводит к появлению волны напряжения, сдвинутой по фазе на 90 ° с волной тока. Глядя на график, кажется, что волна напряжения имеет «фору» по сравнению с волной тока; напряжение «опережает» ток, а ток «отстает» от напряжения.

Ток отстает от напряжения на 90 ° в чисто индуктивной цепи.

Все становится еще интереснее, когда мы строим график мощности для этой цепи:

В чисто индуктивной цепи мгновенная мощность может быть положительной или отрицательной.

Поскольку мгновенная мощность является произведением мгновенного напряжения и мгновенного тока (p = ie), мощность равна нулю, когда мгновенный ток или напряжение равно нулю. Если мгновенный ток и напряжение положительные (над линией), мощность положительная.

Как и в примере с резистором, мощность также является положительной, когда мгновенные ток и напряжение отрицательны (ниже линии).

Однако, поскольку волны тока и напряжения сдвинуты по фазе на 90 °, бывают случаи, когда одна из них положительна, а другая — отрицательна, что приводит к одинаково частым случаям отрицательной мгновенной мощности .

Что такое отрицательная сила?

Но что означает отрицательная мощность ? Это означает, что катушка индуктивности возвращает мощность в цепь, в то время как положительная мощность означает, что она поглощает мощность из цепи.

Поскольку положительные и отрицательные циклы мощности равны по величине и продолжительности во времени, индуктор возвращает обратно в цепь столько же мощности, сколько потребляет в течение полного цикла.

В практическом смысле это означает, что реактивное сопротивление катушки индуктивности рассеивает нулевую полезную энергию, в отличие от сопротивления резистора, который рассеивает энергию в виде тепла.Имейте в виду, это только для идеальных катушек индуктивности, у которых нет сопротивления провода.

Реактивность в зависимости от сопротивления

Противодействие катушки индуктивности изменению тока означает противодействие переменному току в целом, который по определению всегда изменяется по мгновенной величине и направлению.

Это сопротивление переменному току аналогично сопротивлению, но отличается тем, что оно всегда приводит к сдвигу фаз между током и напряжением и рассеивает нулевую мощность.Из-за различий он имеет другое название: реактивное сопротивление . Реактивность по переменному току выражается в омах, как и сопротивление, за исключением того, что его математический символ — X вместо R.

Чтобы быть конкретным, реактивное сопротивление, связанное с катушкой индуктивности, обычно обозначается заглавной буквой X с буквой L в качестве нижнего индекса, например: X L .

Так как напряжение на катушках индуктивности падает пропорционально скорости изменения тока, они будут снижать большее напряжение при более быстром изменении тока и меньшее напряжение при более медленном изменении тока.Это означает, что реактивное сопротивление в Ом для любой катушки индуктивности прямо пропорционально частоте переменного тока. Точная формула для определения реактивного сопротивления выглядит следующим образом:

Если мы подвергнем индуктор 10 мГн воздействию частот 60, 120 и 2500 Гц, он проявит реактивные сопротивления, указанные в таблице ниже.

Реактивное сопротивление индуктора 10 мГн:
Частота (Герцы) Реактивное сопротивление (Ом)
60 3.7699
120 7,5398
2500 157.0796

В уравнении реактивного сопротивления термин «2πf» (все в правой части, кроме L) имеет особое значение. Это количество радиан в секунду, на которое «вращается» переменный ток, если вы представите себе один цикл переменного тока, представляющий вращение полного круга.

радиан — это единица измерения угла: в одном полном круге 2π радиана, так же как в полном круге 360 °.Если генератор переменного тока является двухполюсным, он будет производить один цикл на каждый полный оборот вала, что составляет каждые 2π радиан или 360 °.

Если эту константу 2π умножить на частоту в герцах (циклов в секунду), результатом будет число в радианах в секунду, известное как угловая скорость системы переменного тока.

Угловая скорость в системах переменного тока

Угловая скорость может быть представлена ​​выражением 2πf или ее собственным символом — строчной греческой буквой омега, которая похожа на нашу строчную римскую букву «w»: ω.Таким образом, формула реактивного сопротивления X L = 2πfL также может быть записана как X L = ωL.

Следует понимать, что эта «угловая скорость» является выражением того, насколько быстро колеблются колебания переменного тока, полный цикл равен 2π радиан. Это не обязательно отражает фактическую скорость вала генератора переменного тока.

Если генератор имеет более двух полюсов, угловая скорость будет кратной скорости вала. По этой причине ω иногда выражается в единицах электрических радиан в секунду, а не в (простых) радианах в секунду, чтобы отличить его от механического движения.

Как бы мы ни выразили угловую скорость системы, очевидно, что она прямо пропорциональна реактивному сопротивлению в катушке индуктивности. По мере увеличения частоты (или скорости вала генератора переменного тока) в системе переменного тока катушка индуктивности будет оказывать большее сопротивление прохождению тока, и наоборот.

Переменный ток в простой индуктивной цепи равен напряжению (в вольтах), деленному на индуктивное реактивное сопротивление (в омах), точно так же, как переменный или постоянный ток в простой резистивной цепи равен напряжению (в вольтах), деленному на сопротивление (в Ом).Пример схемы показан здесь:

Индуктивное сопротивление

Фазовые углы

Однако нужно иметь в виду, что здесь напряжение и ток не совпадают по фазе. Как было показано ранее, напряжение имеет фазовый сдвиг + 90 ° по отношению к току. Если мы представим эти фазовые углы напряжения и тока математически в виде комплексных чисел, мы обнаружим, что сопротивление катушки индуктивности току также имеет фазовый угол:

Ток в катушке индуктивности отстает от напряжения на 90 °.

Математически мы говорим, что фазовый угол сопротивления катушки индуктивности току равен 90 °, что означает, что сопротивление катушки индуктивности току является положительной мнимой величиной. Этот фазовый угол реактивного противодействия току становится критически важным при анализе цепей, особенно для сложных цепей переменного тока, где реактивное сопротивление и сопротивление взаимодействуют.

Будет полезно представить любое сопротивление компонента току в терминах комплексных чисел, а не скалярных величин сопротивления и реактивного сопротивления.

ОБЗОР:

  • Индуктивное реактивное сопротивление — это противодействие, которое индуктор предлагает переменному току из-за сдвинутого по фазе накопления и высвобождения энергии в его магнитном поле. Реактивное сопротивление обозначается заглавной буквой «X» и измеряется в омах, как и сопротивление (R).
  • Индуктивное реактивное сопротивление можно рассчитать по следующей формуле: X L = 2πfL
  • Угловая скорость цепи переменного тока — это еще один способ выразить ее частоту в единицах электрических радиан в секунду вместо циклов в секунду.Его символизирует строчная греческая буква «омега» или ω.
  • Индуктивное реактивное сопротивление увеличивается на с увеличением частоты. Другими словами, чем выше частота, тем сильнее она противодействует потоку электронов переменного тока.

СВЯЗАННЫЕ РАБОЧИЕ ЛИСТЫ:

AC Inductive Circuits

Индуктор обычно представляет собой катушку с проволокой, которая создает переменное магнитное поле вокруг нее, когда через нее протекает переменный ток. Индуктивность — это свойство катушки индуктивности, которая противодействует изменению тока.Измеряется в Генри. Из-за этой индуктивности в катушке индуцируется обратная ЭДС при воздействии на нее переменного тока.

Согласно закону Ленца, эта ЭДС противодействует изменению силы тока. Следовательно, приложенное напряжение должно преодолевать только эту обратную ЭДС, потому что в цепи нет сопротивления. Таким образом, приложенное напряжение и обратная ЭДС должны быть равными и противоположными, чтобы ток протекал по цепи.

Поведение цепи переменного тока с индуктором полностью отличается от цепи постоянного тока.При этом ток, протекающий через катушку, зависит не только от индуктивности, но и от частоты переменного источника. Обсудим вкратце поведение цепи переменного тока при индуктивных нагрузках.

Переменный ток, приложенный через чистый индуктор

Чистый индуктор не имеет сопротивления в обмотке катушки, но имеет только индуктивность. Это свойство индуктивности проявляется во всех двигателях, трансформаторах и генераторах (с некоторым сопротивлением в катушке). На рисунке ниже показана чисто индуктивная цепь с источником переменного напряжения и соответствующие формы сигналов.

Пусть приложенное напряжение, v = V m sin ωt. Как указано выше, индуцированная ЭДС равна приложенному напряжению и противоположна ему, т.е. v = — e

Где e — обратная ЭДС и равна –L di / dt

Подставляя выражение для ЭДС, получаем

v = L di / dt

V m sin ωt = L di / dt

di = (V m / L) sin ωt dt

Применяя интегрирование с обеих сторон, мы получаем

i = ( V м / L) ∫ sin ωt dt

= (V m / ωL) (- cos ωt)

i = (V m / wL) (sin ωt — π / 2)

Когда (sin ωt — π / 2) равно единице, ток, протекающий по цепи, будет максимальным.Таким образом,

im = (V m / ωL)

Тогда текущее уравнение принимает вид

i = i m sin (ωt — π / 2)

Где i m = (V m / ωL)

Из приведенных выше выражений для тока и напряжения ясно, что ток отстает от напряжения на 900. Следовательно, в чистой индуктивной цепи ток находится в квадратуре с напряжением, как показано на диаграммах на приведенном выше рисунке.

Это означает, что когда изменение тока является максимальным (при токе, проходящем через ноль), напряжение, индуцированное на катушке индуктивности, является максимальным.Точно так же при максимальных значениях тока, когда ток не изменяется, индуцированное напряжение на катушке индуктивности будет равно нулю.

Таким образом, напряжение на катушке индуктивности опережает ток через эту катушку индуктивности на (четверть) цикла. Векторная диаграмма чисто индуктивной цепи переменного тока приведена ниже.

Индуктивное реактивное сопротивление

Из приведенного выше вывода уравнение максимального тока дается как

i м = (V m / ωL)

ωL = V m / i m

Это отношение напряжения к току является противодействием индуктивной цепи протеканию тока.Эта величина wL называется индуктивным реактивным сопротивлением и обозначается XL и измеряется в Ом.

Индуктивное сопротивление цепи переменного тока может быть представлено как

XL = ωL = 2ΠfL (так как ω = 2Πf)

Где XL — индуктивное реактивное сопротивление в Ом

f — частота напряжения питания

L является индуктивность катушки в Генри

Вышеприведенное уравнение говорит, что, когда частота входного питания увеличивается, скорость, с которой изменяется ток, также изменяется.Следовательно, наведенная ЭДС (или напряжение реакции) на катушке индуктивности будет увеличиваться.

В результате чистый ток, протекающий через катушку индуктивности, будет уменьшен. Следует сделать вывод, что реактивное сопротивление катушки индуктивности линейно зависит от частоты источника питания, как показано на рисунке.

Мощность и коэффициент мощности в индуктивной цепи переменного тока

Мощность в цепи переменного тока является произведением мгновенного напряжения и тока. Это можно представить как

P = v × i

P = V m sin ωt × I m sin (ωt — 90)

При интегрировании по циклу получаем,

P = V m sin ωt × I м sin (ωt — 90)

P = 1 / 2π (∫ 0 V м sin ωt × I м sin (ωt — 90) dωt)

= (V m I m / 2π) (∫ 0 sin ωt × (- cos ωt) dwt)

= (V m I m / 2π) (∫ 0 (- sin 2 ωt) / 2 dwt)

= (V m I m / 8π) (cos 4π — cos 0)

= (V m I m / 8π ) (1 — 1)

P = 0

Средняя мощность в чистой катушке индуктивности всегда равна нулю, поскольку количество энергии, полученное от источника за полупериод, возвращается к источнику в следующем полупериоде.

На рисунке ниже показана кривая мощности индуктивной цепи переменного тока, в которой положительная мощность равна отрицательной мощности, поэтому результирующая мощность за цикл равна нулю. Это ясно объясняет, что чистая индуктивность не потребляет энергии.

В этой цепи ток также синусоидальный, но отстает от напряжения на 900. Поскольку ток отстает от напряжения на 90 0 , разность фаз θ равна 90 0 . Тогда

Коэффициент мощности, cos 90 = 0

Коэффициент мощности в чисто индуктивной цепи равен нулю, т.е.е., чисто отстающий коэффициент мощности.

Series RL Circuit

Как мы знаем, физической цепи чистой индуктивности не существует, потому что каждая катушка наряду с индуктивностью имеет некоторое сопротивление обмотки. В такой схеме сопротивление считается последовательным элементом индуктора.

Рассмотрим рисунок ниже, на котором чистое сопротивление последовательно соединено с чистой индуктивностью. Эта последовательная комбинация подключена к источнику переменного тока с напряжением v = V m sin ωt.

В цепи последовательно R L напряжение на катушке индуктивности не совпадает по фазе с током, протекающим по цепи, и напряжением на сопротивлении, как показано на рисунке выше.Индуцированное напряжение в катушке индуктивности препятствует прохождению тока, и, следовательно, V L опережает ток I и падает на сопротивлении V R на 90 0 .

Пусть I — ток, протекающий по цепи, V L и V R — падения напряжения на индуктивности и сопротивлении соответственно.

Напряжение на резисторе, В R = I R

Напряжение на катушке индуктивности, В L = I × XL (где XL = 2πfL)

Из приведенного выше вектора

В = √ (V R 2 + V L 2 ) = √ (IR) 2 + (I XL) 2 )

= I √ (R 2 + XL 2 ) = I Z

Где Z — полное сопротивление в последовательной цепи R L , равное √ (R 2 + XL 2 ).

Треугольник полного сопротивления

Сопротивление, создаваемое цепью переменного тока потоку синусоидального тока, называется импедансом. Его также можно определить как отношение синусоидального напряжения к току. Обозначается буквой Z и измеряется в Ом.

Из векторной диаграммы серии RL,

tan ϕ = V L / V R = XL / R

cos ϕ = V R / V = ​​R / Z

sin ϕ = V L / V = ​​XL / Z

Если все стороны треугольника, полученного в последовательной цепи X L , разделить на ток, мы получим треугольник импеданса, как показано на рисунке.Из этого треугольника компоненты R, X L и Z могут быть выражены как

R = Z cos ϕ

XL = Z sin ϕ

Z = √ (R 2 + XL 2 )

And ϕ = tan-1 (XL / R)

Пример

Найдите выражение для тока, а также рассчитайте мощность последовательной цепи RL с R = 50 Ом и L = 0,159 Гн, возбуждаемой напряжением v = 283 sin 100πt .

Индуктивное реактивное сопротивление, XL = 2πfL = 100π × 0,159

= 49,95 Ом

Z = R + j XL = 50 + j49.95

Преобразуя в полярную форму, получаем Z = 70,675 ∠44,97 Ом

Ток, i = v / Z = (283 sin (100πt — 44,97)) / 70,675

i = 4 sin (100πt — π / 4) A

P = VI cos θ

= (283 / √2) (4 / √2) cos 44,97

= 400,43 A

Какое активное сопротивление цепи. НАПРИМЕР. Воропаев Электротехника

Сопротивление цепи переменного тока, в которой происходит преобразование электрической энергии в полезную работу или в тепловую энергию, называется активным сопротивлением.К активным сопротивлениям промышленной частоты (50 Гц) относятся, например, электрические лампы накаливания и электронагревательные устройства. Рассмотрим цепь переменного тока, которая включает активное сопротивление. в цепи переменного тока с активным сопротивлением при изменении величины и направления напряжения пропорционально одновременно меняются величина и направление тока. Это означает, что ток и напряжение совпадают по фазе. Строим векторную диаграмму текущих значений тока и напряжения для цепи с активным сопротивлением.Для этого вектор напряжения U фиксируется в выбранном масштабе по горизонтали. Чтобы показать на векторной диаграмме, что напряжение и ток в цепи синфазны ( = 0), переносим вектор тока I в сторону вектора напряжения. Сила тока в такой цепи определяется по закону Ома: I = U / R.


Билет 20. Вопрос 1. Переменный ток: понятие, получение, характеристики, единицы измерения.

Переменный ток — электрический ток, периодически меняющий модуль и направление.Для передачи и распределения электроэнергии в основном используется Переменный ток из-за простоты его преобразования напряжения практически без потери мощности. Генераторы и двигатели Переменный ток по сравнению с машинами постоянного тока равной мощности, они меньше по размеру, проще по конструкции, надежнее и дешевле. Переменный ток можно выпрямить, например, с помощью полупроводниковых выпрямителей, а затем снова преобразовать в полупроводниковые инверторы в Переменный ток другую, регулируемую частоту; это позволяет использовать простые и дешевые бесщеточные двигатели. Характеристики переменного тока. Средняя мощность переменного тока за период T равна: P cf. = I m * U m cos () / 2, где  — фазовый сдвиг между током и напряжением, U m и I m — максимальные (амплитудные) значения напряжения и тока. В цепи переменного тока с активной нагрузкой колебания тока совпадают по фазе с колебаниями напряжения. Если U = U m sin (wt), то I = I m sin (wt) и cos () = 1. Действующие (действующие) значения тока и напряжения рассчитываются по формулам: I d = I м / корень 2, U д = U м / корень 2.

Билет 21. Вопрос 1. . Режимы работы трансформатора: холостой ход, рабочий режим, режим короткого замыкания. КПД трансформатора. Режим холостого хода трансформатора называется рабочим режимом, когда одна из обмоток трансформатора запитана от источника переменного напряжения, а другие обмотки разомкнуты. Настоящий трансформатор может иметь этот режим работы, когда он подключен к сети, а нагрузка, питаемая вторичной обмоткой, еще не включена. Режим работы — это работа трансформатора при подключении потребителей или под нагрузкой (под нагрузкой понимается ток вторичной цепи — чем он больше, тем больше нагрузка).К трансформатору подключаются различные типы потребителей: электродвигатели, осветительные приборы и т. Д. Режим короткого замыкания, , возникающий случайно при работе на номинальном первичном напряжении, является аварийным процессом, сопровождающимся очень большими токами в обмотках. Многократное увеличение токов по сравнению с номинальными (в 10-20 раз) может привести к повреждению изоляции обмоток в результате нагрева и к разрушению обмоток механическими силами, возникающими в этом режиме между обмотками.КПД трансформатора — это отношение активной мощности, передаваемой нагрузке, к активной мощности, подаваемой на трансформатор. КПД трансформатора имеет большое значение. У силовых трансформаторов малой мощности она составляет около 0,95, а у трансформаторов мощностью в несколько десятков тысяч киловольт-ампер достигает 0,995. Определение КПД по формуле с использованием непосредственно измеренных мощностей P1 и P2 дает большую ошибку. Эту формулу удобнее представить в другом виде: КПД = P 2 / P 1 + общая дельта R.


Билет 22. Вопрос 1. Соединение фаз генератора и потребителей треугольником: симметричная и несимметричная нагрузка, векторная диаграмма.

ABC — начало фазы, хуz — конец фазы, AA ’- линейный провод. При соединении треугольником начало фазы соединяется с концом предыдущей фазы и смещается на 120 градусов. при симметричной нагрузке, соединенной треугольником, линейный ток в √3 раз больше фазного тока. Il = root 3> Если.Ул = УФ. В трехфазных цепях различают симметричные (сопротивление в каждой фазе нагрузки одинаково) и асимметричную (сопротивление нагрузки хотя бы в одной фазе различается) нагрузок. При симметричной нагрузке достаточно исследовать одну фазу и все умножить на 3. Для асимметричной нагрузки необходимо исследовать каждую фазу, а затем сложить ее. При симметричной нагрузке фазные напряжения отдельных фаз равны между собой. При несимметричной нагрузке трехфазной системы симметрия токов и напряжений нарушается.Однако в четырехпроводных схемах небольшой асимметрией фазных напряжений часто пренебрегают. В этих случаях существует взаимосвязь между линейным и фазным напряжениями: U l = sqrtU f.

P = корень 3 U l I l cosphi = 3U f I f COSphi. BT; Q = sqrt3 U l I l SINfi = 3U f I f SINfi. VAR

S = sqrt3 U l I l = 3U f I f VA

Лампы накаливания, печи сопротивления, бытовое тепло

Пассивные и активные электронные компоненты | Глава 3 — Электропитание в системах переменного тока

На предыдущей странице мы обсуждали векторов, но мы не исследовали в полной мере значение методов векторов для анализа цепей переменного тока.Фундаментальный момент, который необходимо понять, заключается в следующем: представляя параметры цепи с помощью комплексных чисел, мы даем возможность анализировать цепь переменного тока, как если бы это была цепь постоянного тока.

От сопротивления к сопротивлению

Мы уже знаем, что токи и напряжения в цепи переменного тока могут быть записаны в виде векторов, т. Е. Как величина (соответствующая пиковому значению синусоидальной формы волны), сопровождаемая углом (соответствующим разности фаз между синусоидальной формой волны и эталонный сигнал).Фазор также можно записать как комплексное число, состоящее из действительной и мнимой частей.

В случае токов и напряжений представление величины плюс угол более интуитивно понятно. Однако представление «действительное плюс мнимое» полезно для дополнительного и очень важного применения векторов: представления импеданса .

В базовой цепи постоянного тока у нас есть постоянное напряжение, постоянный ток и сопротивление. В цепи переменного тока у нас есть переменное напряжение, переменный ток и полное сопротивление.Импеданс указывает на сопротивление компонента току, так же как и сопротивление, но он также передает фазовый сдвиг, связанный с компонентом. Другой способ объяснить это: сопротивление устанавливает взаимосвязь между величиной напряжения на компоненте и величиной тока, протекающего через компонент. Импеданс устанавливает соотношение между величиной напряжения и тока, а также разностью фаз между напряжением и током.

Импеданс записывается как комплексное число и обозначается буквой Z, и в контексте анализа цепей значения импеданса полностью эквивалентны сопротивлениям. Это означает, что методы анализа, которые мы используем для цепей постоянного тока , могут быть применены непосредственно к цепям переменного тока , если мы представим компоненты как импедансы, а токи и напряжения как векторы.

Рис. 1. Фазорная нотация и понятие импеданса позволяют нам анализировать цепи переменного тока, как если бы они были цепями постоянного тока (хотя математика при анализе переменного тока часто бывает более сложной).

Импеданс пассивных компонентов

На данный момент мы знаем, что компонент может быть представлен как импеданс, но каков именно импеданс резистора, конденсатора или катушки индуктивности?

Для резистора все просто: импеданс равен сопротивлению.

Обычно нам не нужно писать «j0»; в данном случае он включен, чтобы подчеркнуть тот факт, что сопротивление чисто реально и, следовательно, импеданс резистора не имеет мнимой части.

Импеданс конденсатора и катушки индуктивности выражается следующим образом:

Как вы, возможно, помните, ω обозначает угловую частоту и равна 2π умноженной на «нормальную» частоту (т.е. частоту, измеренную в циклах в секунду). Кроме того, к этим выражениям прилагается единица измерения в омах. Сопротивление, реактивное сопротивление и импеданс — все они передают информацию о сопротивлении току, и все они используют одну и ту же единицу.

В следующих разделах обсуждается очень важная информация, заключенная в этих уравнениях.

Частотная характеристика

В выражении для емкостного импеданса член ω стоит в знаменателе формулы импеданса. В выражении для индуктивного импеданса индуктивность умножается на член ω. Эти отношения показывают фундаментальное поведение конденсаторов и катушек индуктивности в контексте цепей переменного тока: конденсаторы оказывают меньшее сопротивление току при увеличении частоты, а катушки индуктивности обеспечивают большее сопротивление потоку тока при увеличении частоты.Вот почему мы используем конденсаторы для блокировки постоянной составляющей сигнала и катушки индуктивности для подавления высокочастотного шума.

Сопротивление и реактивное сопротивление

Действительная часть импеданса указывает величину сопротивления, а мнимая часть указывает величину реактивного сопротивления. Резистор — это чисто резистивный компонент, поэтому его импеданс имеет только реальную часть. Конденсаторы и катушки индуктивности, с другой стороны, являются чисто реактивными компонентами, и, как вы можете видеть, для обоих этих компонентов полное выражение импеданса умножается на j, что означает, что импеданс имеет только мнимую часть.

Рассеивание мощности и фазовый сдвиг

Действительная часть импеданса соответствует той части компонента или схемы, которая создает рассеяние мощности, а мнимая часть соответствует части, которая создает разность фаз между напряжением и током. Чисто воображаемый импеданс (то есть реактивное сопротивление) не рассеивает мощность, а чисто реальный импеданс (то есть сопротивление) не вызывает сдвига фазы.

Эти утверждения согласуются с выражениями, показанными выше: сопротивление резистора не имеет реактивного сопротивления; весь ток, проходящий через резистор, способствует рассеиванию мощности, и ток остается в фазе с напряжением.Импеданс конденсатора или катушки индуктивности имеет реактивное сопротивление, но не сопротивление; эти компоненты накапливают энергию, а не выделяют ее в окружающую среду, и они создают разность фаз между током и напряжением.

Емкостный и индуктивный фазовый сдвиг

Мы знаем, что конденсаторы и катушки индуктивности влияют на соотношение фаз между напряжением и током, но какой именно фазовый сдвиг они вносят?

Мы можем найти фазовый сдвиг комплексного импеданса, используя полярную форму вместо действительной и мнимой частей; угол представления полярной формы — это фазовый сдвиг.Как правило, этот угол вычисляется с помощью функции arctan, но в этом случае мы просто посмотрим на импеданс в комплексной плоскости:

Рис. 2. Векторы представляют собой (чисто мнимое) полное сопротивление конденсатора и катушки индуктивности.

Емкостной импеданс простирается вдоль отрицательной мнимой оси, что соответствует углу –90 °; индуктивный импеданс простирается вдоль положительной мнимой оси, что соответствует углу + 90 °.Таким образом, как конденсаторы, так и катушки индуктивности создают разность фаз 90 ° между напряжением и током.

Чтобы понять значение положительного и отрицательного знаков, примите во внимание следующее: Чтобы вычислить напряжение по закону Ома, мы умножаем ток на сопротивление. Если мы распространим это на область цепей переменного тока, мы умножим вектор, представляющий ток, на полное сопротивление компонента.

Когда мы умножаем комплексные числа в полярной форме, мы должны складывать значения углов. Мы знаем, что угол импеданса конденсатора составляет –90 °, а угол импеданса катушки индуктивности составляет + 90 °.Таким образом, напряжение на конденсаторе будет иметь фазу, равную фазе тока минус 90 °, а напряжение на катушке индуктивности будет иметь фазу, равную фазе тока плюс 90 °. Это согласуется с тем, что мы узнали ранее в этой главе: в емкостной цепи напряжение отстает от тока; в индуктивной цепи напряжение ведет к току.

Далее: Мощность в реактивных цепях

Мы изучили важную информацию, относящуюся к импедансу, который является фундаментальной концепцией, которая часто встречается при проектировании различных типов электрических систем.На следующей странице мы рассмотрим некоторые дополнительные сведения, касающиеся мощности в реактивных цепях.

Активная, реактивная и полная мощность в цепях переменного тока

Активная мощность:

Если активное сопротивление (например, нагревательный элемент) подключено к цепи переменного тока, результирующие напряжение и ток совпадают по фазе (синяя и красная кривые на схеме ниже). Умножение связанных пар мгновенных значений напряжения и тока дает мгновенную мощность (зеленая кривая).

Такая кривая мощности всегда положительна, потому что для активного сопротивления напряжение и ток всегда либо положительные, либо отрицательные. Положительная мощность передается от генератора к потребителю. Зеленые зоны отображают выполненную активную работу. Поскольку мощность имеет частоту в два раза превышающую частоту напряжения или тока, не может быть нанесено на график вместе с током и напряжением на нормальной векторной диаграмме.

Питание переменного тока p ( t ) имеет пиковое значение p 0 = u 0 · i 0 и может быть преобразовано путем приравнивания областей под кривой в эквивалентную мощность постоянного тока , или активной мощности P .Активная мощность для активного сопротивления составляет половину пиковой мощности, т.е.

Другими словами:

Активная мощность для активного сопротивления является произведением среднеквадратичного напряжения и действующего тока.

Реактивная мощность:

Если чистое реактивное сопротивление, т. Е. Емкостное или индуктивное сопротивление, подключено к цепи переменного тока, смещение фаз j между током и напряжением составляет 90 °, ток опережает напряжение в случае емкости и отстает от напряжения в случай индуктивности (как показано на диаграмме ниже).Кривая мощности здесь симметрична относительно оси времени, так что положительная и отрицательная (серые) области компенсируют друг друга, и в целом активная мощность не потребляется. Отрицательные значения означают, что мощность возвращается от потребителя к генератору. В течение одного периода энергия дважды возвращается от катушки (потребителя) к генератору. Общая энергия постоянно колеблется между генератором и потребителем. Это приводит к чистому потреблению реактивной мощности индуктивного или емкостного характера в зависимости от используемого компонента.Реактивная мощность обозначается Q и выражается в единицах Var .

Полная мощность:

Если нагрузка, включающая компоненты активного и реактивного сопротивления, подключена к переменному напряжению, возникают компоненты активной и реактивной мощности. Схема ниже демонстрирует это в случае индуктивной нагрузки, ток и напряжение которой сдвинуты по фазе на 60 °. Кривая мощности здесь в основном расположена выше оси времени.Серые области частично компенсируют друг друга и представляют компонент реактивной мощности, а зеленые области представляют активную мощность (или выполненную активную работу).

Умножение измеренных значений напряжения и сдвинутого по фазе тока дает здесь полную мощность S , которая выражается в вольт-амперах (ВА):

Полная мощность — это , а не мера преобразования электрической энергии в цепи, вместо этого она служит просто вычисляемой переменной, состоящей из активной и реактивной мощности.Соответственно, активная мощность P , показанная измерителем мощности (ваттметром) при наличии фазового сдвига между током и напряжением, всегда меньше, чем полная мощность S , рассчитанная на основе действующего значения тока и напряжения.

Активное сопротивление в цепи переменного тока

Электрическая цепь предполагает наличие в своем составе ряда различных компонентов. Конденсаторы и элементы индуктивности включены в схему для получения разного эффекта.Сопротивления присутствуют в виде отдельных элементов-резисторов и резисторов соединения проводов. С физикой влияние компонентов схемы на законы протекания электрических явлений было изучено достаточно тщательно и мало отличается от природы протекания электрического тока, будь то под влиянием переменного или постоянного напряжения.

Сопротивление в цепи переменного тока работает так же, как и в режиме постоянного напряжения потока. Другое дело — элементы емкости и индуктивности.Если в подсхеме, в которой подключен конденсатор, отсутствует постоянная составляющая тока, то на индуктивность она никак не влияет. Прохождение постоянного тока в индуктивности влияет только на активное сопротивление катушки.

Совершенно иная ситуация в описании электрических процессов в цепи переменного тока. Конденсаторы становятся проводниками, а индукторы (дроссель, трансформатор и т. Д.) Находят индуктивное сопротивление, которое, в свою очередь, играет гораздо более важную роль, а сопротивление часто просто не принимается во внимание.

Но, тем не менее, необходимость точных расчетов требует учитывать эту составляющую. Чтобы начать понимать, как активное сопротивление катушки сочетается с индуктивным, следует в целом рассматривать устройство как стандартный индуктор.

Являясь составной частью электрической цепи, данное устройство является не чем иным, как биполярным элементом множества электрических, электромеханических и электронных систем и устройств. В качестве основного параметра используется значение самоиндукции.Это, в свою очередь, зависит только от геометрических размеров и материалов изготовления. На величину индуктивности не влияет ни сила тока, ни напряжение. Использование индукторов в фильтрах для подавления помех, их использование позволяет добиться сглаживания, также катушка может накапливать энергию, что широко используется при проектировании резонансных цепей.

Чтобы проанализировать, какое влияние оказывает сопротивление работе дроссельной заслонки, вы должны рассмотреть цепь переменного тока с одним элементом, которым является индуктор.Рассчитывая активное и индуктивное сопротивление, казалось бы, самый простой способ вычислить общие значения — это сумма их модулей.

Но с такими количествами дело обстоит не так просто, как могло бы показаться. Опуская теоретическое обоснование, опишите использование на практике метода суммирования активного и индуктивного сопротивлений.

Чтобы найти общий балл, постройте прямоугольный треугольник. Одна сторона — это сопротивление, а другая — индуктивная. Гипотенуза будет равна сопротивлению цепи, которое согласно определению равно квадратному корню из суммы квадратов активного и индуктивного сопротивлений.

Выполненные таким образом расчеты дают более точную информацию о процессах, происходящих в цепи переменного тока, элементами которой являются индукторы. В формуле закона Ома есть возможность использовать величину импеданса. В будущем следует отметить, что значительное сопротивление по сравнению с индуктивным может влиять на фазовый сдвиг между током и напряжением. Поэтому при изготовлении катушек индуктивности широко используется конструкция сердечника, которая дает значительные преимущества по индуктивному сопротивлению.

Импеданс в цепях переменного тока — Видео и стенограмма урока

Импеданс и резисторы

Понятие сопротивления, которое мы обсуждали в другом уроке, также применимо к цепям переменного тока, но немного по-другому. В цепях переменного тока сопротивление изменяется в зависимости от того, в какой части синусоидальной кривой вы находитесь в данный момент. Чтобы отразить это, мы используем другой термин: импеданс. Импеданс — это сопротивление компонента в цепи переменного тока.Это то, что вы получите, если воспользуетесь среднеквадратичными значениями тока и напряжения в законе Ома. Мы просто заменяем R сопротивления на Z для импеданса.

I = V / R становится…

Irms = Vrms / Z

Импеданс, как и сопротивление, измеряется в Ом.

Решение проблем с использованием комбинаций резисторов, конденсаторов или катушек индуктивности может быть затруднено в цепях переменного тока и требует использования комплексных чисел.

Реактивное сопротивление конденсаторов и индукторов

Как будто у нас не хватило названий для одного и того же, вот еще одно: реактивное сопротивление .Говоря конкретно о конденсаторах и катушках индуктивности, их «импеданс» обычно называют «реактивным сопротивлением». Для индуктора это называется индуктивным реактивным сопротивлением и используется символ XL, а для конденсатора это называется емкостным реактивным сопротивлением и используется символ XC. Еще раз, это можно вернуть в обычное уравнение закона Ома, заменив сопротивление R.

I = V / R становится…

Irms = Vrms / XL (индуктивное реактивное сопротивление)

или

Irms = Vrms / XC (емкостное реактивное сопротивление)

Это уравнение, которое мы можем использовать для расчета емкостного реактивного сопротивления, где C — емкость конденсатора, измеренная в Фарадах, а f — частота источника переменного тока — количество раз, когда он меняет направление в секунду. — измеряется в Герцах.

XC = 1/2 * pi * f * C

И это уравнение для индуктивного реактивного сопротивления, где L — индуктивность катушки индуктивности, измеренная в Генри, а f — снова частота источника переменного тока, измеренная в Фарады.

XL = 2 * pi * f * L

Пример расчета

Итак, например, предположим, что у вас есть цепь переменного тока, которая содержит индуктор 5 Генри на основном источнике питания на 120 вольт. И вас просят рассчитать среднеквадратичное значение тока, протекающего по цепи, если частота переменного тока составляет 60 Гц.

Ну, прежде всего, мы должны записать то, что мы знаем. Индуктивность L составляет 5 Генри. Напряжение, В, составляет 120 вольт. А частота f равна 60 Гц.

Нас просят вычислить действующее значение тока Irms. Основываясь на законе Ома, Irms равно Vrms, деленному на R. Но поскольку это цепь переменного тока, это действительно Vrms, деленное на Z (импеданс), и поскольку мы говорим об индукторе, мы должны снова заменить это, сделав это Vrms, разделенное на XL.

Irms = Vrms / XL

Мы знаем, что Vrms равно 120 вольт, поэтому нам просто нужно вычислить XL.К счастью, у нас есть уравнение для этого: XL равен 2 * pi * f * L. Итак, 2 умножить на пи, умножить на частоту 60 Гц, умножить на индуктивность (5 Генри). Введите все это в калькулятор, и вы получите индуктивное реактивное сопротивление (XL) 1885 Ом.

Наконец, подставьте ЭТО число в уравнение закона Ома, 120, деленное на 1885, дает нам ток 0,064 ампера.

Вот и все, готово.

Краткое содержание урока

Большинство электрических цепей в доме являются цепями переменного тока. Переменный ток — это то место, где вместо тока, протекающего только в одном направлении по цепи, он очень быстро переключает направление. В цепи переменного тока ток, напряжение и мощность изменяются синусоидально — в виде синусоидальной кривой. Поскольку ток и напряжение всегда меняются, мы используем то, что называется среднеквадратичным (или среднеквадратичным) током и среднеквадратичным напряжением. Вы можете рассчитать эти числа, используя следующие уравнения:

Irms = I0 / квадратный корень из 2

Vrms = V0 / квадратный корень из 2

V0 — пиковое или максимальное напряжение, измеренное в вольтах, а I0 — пиковый или максимальный ток. измеряется в амперах.

В цепях переменного тока мы не используем термин «сопротивление». Вместо этого мы называем это импедансом: Импеданс — это просто сопротивление компонента в цепи переменного тока. Это то, что вы получите, если воспользуетесь среднеквадратичными значениями тока и напряжения в законе Ома. Мы просто заменяем R сопротивления на Z для импеданса.

I = V / R становится…

Irms = Vrms / Z

Когда у нас есть конденсаторы или катушки индуктивности в цепи, мы снова заменяем Z на импеданс. Мы называем сопротивление конденсатора или катушки индуктивности реактивным сопротивлением. Реактивное сопротивление — это просто импеданс конденсатора или катушки индуктивности. Для индуктора это называется индуктивным реактивным сопротивлением и используется символ XL, а для конденсатора это называется емкостным реактивным сопротивлением и используется символ XC. Опять же, это можно вернуть в обычное уравнение закона Ома, заменив сопротивление R.

I = V / R становится…

Irms = Vrms / XL для индуктивного реактивного сопротивления

или

Irms = Vrms / XC для емкостное реактивное сопротивление

У нас также есть уравнение для расчета емкостного реактивного сопротивления и уравнение для расчета индуктивного реактивного сопротивления.В этих уравнениях C — это емкость конденсатора, измеренная в фарадах, L — индуктивность индуктора, измеренная в Генри, а f — частота источника переменного тока — количество раз, когда он меняет направление в секунду, измеряется в Герц.

XC = 1/2 * pi * f * C

XL = 2 * pi * f * L

Результаты обучения

После этого урока вы сможете:

  • Описывать, как работают цепи переменного тока
  • Напомним синусоидальный характер цепей переменного тока
  • Объясните, что такое импеданс и как его рассчитать
  • Определите уравнения для расчета емкостного и индуктивного реактивного сопротивления
.

alexxlab

Добавить комментарий

Ваш адрес email не будет опубликован. Обязательные поля помечены *